Limit of sin(1/x) as x approaches infinity: Understanding the Concept

  • Thread starter Thread starter rambo5330
  • Start date Start date
Click For Summary
The limit of sin(1/x) as x approaches infinity is zero, as 1/x approaches zero and sin(0) equals zero. A misunderstanding arose when a teacher initially suggested the limit equals one, possibly confusing it with the limit of sin(x)/x as x approaches zero. The application of the squeeze theorem confirms that lim(x->inf) sin(1/x)/x equals zero, while lim(x->0) sin(1/x) does not exist due to oscillation. Clarification from the teacher acknowledged the correct limit but caused further confusion among students. Ultimately, the correct understanding is that lim(x->inf) sin(1/x) equals zero.
rambo5330
Messages
83
Reaction score
0

Homework Statement


lim (x->inf) sin(1/x)

i have a teacher that seems to think this is equal to 1.. I don't see how this is correct
as x approaches infinity 1/x aproaches zero... sin 0 = 0 right?
or is this the wrong way of thinking?


Homework Equations





The Attempt at a Solution

 
Physics news on Phys.org
You're right. Perhaps your teacher was thinking of

\lim_{x \to 0} \frac{\sin x}{x} = 1
 
Thanks very much yes maybe he was thinking of that. but here is the actuall email he sent out correcting himself... if anything it confused me more... does what he say hold true?
"
The application of the squeeze theorem that we did in class to lim(x->inf)sin(1/x)/x, was correct and gives the answer 0, but I was wrong to say that lim(x->inf)sin(1/x) DNE, it is as some students saw 1. What I intended was to consider the limit: lim(x->0) x sin(1/x). Now lim(x->0)sin(1/x) = DNE because as x->0, sin(1/x) oscillates wildly between -1 an +1, so ones really needs the squeeze theorem here!"
 
nop 1/x --->0 so sin(1/x) goes to zero
 
well if u know L'hospital rule you can use it on <br /> \lim_{x \to 0} \frac{\sin x}{x} = 1<br />

so u get <br /> \lim_{x \to 0} \frac{\cos x}{1} = 1<br />
 
Question: A clock's minute hand has length 4 and its hour hand has length 3. What is the distance between the tips at the moment when it is increasing most rapidly?(Putnam Exam Question) Answer: Making assumption that both the hands moves at constant angular velocities, the answer is ## \sqrt{7} .## But don't you think this assumption is somewhat doubtful and wrong?

Similar threads

Replies
5
Views
2K
Replies
2
Views
1K
  • · Replies 5 ·
Replies
5
Views
2K
  • · Replies 13 ·
Replies
13
Views
3K
  • · Replies 23 ·
Replies
23
Views
2K
  • · Replies 8 ·
Replies
8
Views
2K
Replies
4
Views
4K
  • · Replies 15 ·
Replies
15
Views
7K
Replies
3
Views
2K
  • · Replies 3 ·
Replies
3
Views
2K